Last visit was: 23 Apr 2024, 13:31 It is currently 23 Apr 2024, 13:31

Close
GMAT Club Daily Prep
Thank you for using the timer - this advanced tool can estimate your performance and suggest more practice questions. We have subscribed you to Daily Prep Questions via email.

Customized
for You

we will pick new questions that match your level based on your Timer History

Track
Your Progress

every week, we’ll send you an estimated GMAT score based on your performance

Practice
Pays

we will pick new questions that match your level based on your Timer History
Not interested in getting valuable practice questions and articles delivered to your email? No problem, unsubscribe here.
Close
Request Expert Reply
Confirm Cancel
SORT BY:
Date
Tags:
Show Tags
Hide Tags
User avatar
Senior Manager
Senior Manager
Joined: 29 Jul 2006
Posts: 379
Own Kudos [?]: 1268 [28]
Given Kudos: 0
Send PM
Most Helpful Reply
Math Expert
Joined: 02 Sep 2009
Posts: 92883
Own Kudos [?]: 618583 [1]
Given Kudos: 81563
Send PM
General Discussion
User avatar
Manager
Manager
Joined: 24 Sep 2006
Posts: 127
Own Kudos [?]: 144 [0]
Given Kudos: 0
Send PM
avatar
Manager
Manager
Joined: 01 Mar 2005
Posts: 51
Own Kudos [?]: 3 [0]
Given Kudos: 0
Send PM
Economist: Some policymakers believe that our country's continued econ [#permalink]
AimHigher wrote:
Whats wrong with D??


D says that the author is specifically criticizing the backers of the proposal by pointing out other programs where their judgement has been wrong.

In the passage, the author points out that programs similar to the one proposed have been wrong - he says nothing about other programs the backers have supported in the past being wrong.

It's the difference between saying: "The backers support X. But X has been wrong in the past, so X is wrong now" and "The backers support X. But the backers have been wrong about Y and Z in the past, so they must be wrong about X."

I think the answer is A. The author is challenging the idea that offering the plan will increase the amount of savings. What is the OA?
User avatar
Manager
Manager
Joined: 22 Feb 2007
Posts: 97
Own Kudos [?]: 45 [0]
Given Kudos: 0
Send PM
Re: Economist: Some policymakers believe that our country's continued econ [#permalink]
I'll go with A

The backers of Legislative proposal base their support to the proposal based on a premise that implementation of the proposal would increase the amount of money available for banks to loan at a relatively small cost to the government in lost tax revenues.

The economist cites a specific example where such effect has not been observed. Thus, he questions the premise on which the proposal is based.

The economist does not directly question the judgment of the proposal’s backers. Hence D cannot be the answer.

I found this one tough... :?
User avatar
Manager
Manager
Joined: 11 Feb 2007
Posts: 193
Own Kudos [?]: 1012 [0]
Given Kudos: 0
Send PM
Re: Economist: Some policymakers believe that our country's continued econ [#permalink]
I would go with A.

The purpose of the proposal is to increase the amount of personal savings (i.e. banks have more money to lend) but the author at the end refutes that a similar proposal (with seemingly similar purpose) failed because "the overall level of personal savings was unchanged."

Although it did not explicity say that the similar proposal in the past was a "failure", we can infer that it did not achieve its goal from the use of the word "yet" to introduce it.
Manager
Manager
Joined: 05 Oct 2016
Posts: 55
Own Kudos [?]: 161 [0]
Given Kudos: 11
Location: China
Concentration: Healthcare, Entrepreneurship
WE:Sales (Health Care)
Send PM
Re: Economist: Some policymakers believe that our country's continued econ [#permalink]
OA is A I also go with D but after reread the D I found the author didn't question the judgement of the backers so I think that's the reason A is correct.
Manager
Manager
Joined: 17 Oct 2015
Posts: 136
Own Kudos [?]: 282 [0]
Given Kudos: 453
Location: India
Concentration: Finance
Schools: ISB '21 (A)
GMAT 1: 690 Q47 V37
GMAT 2: 700 Q44 V41
WE:Corporate Finance (Investment Banking)
Send PM
Re: Economist: Some policymakers believe that our country's continued econ [#permalink]
Even though OA is A I am confused here because the author is not challenging a premise... He is challenging an underlying assumption of policymakers that Tax incentives are enough to make people save more. nightblade354 GMATNinja sir please share your views..
Current Student
Joined: 20 Oct 2018
Posts: 184
Own Kudos [?]: 127 [0]
Given Kudos: 57
Location: India
GMAT 1: 690 Q49 V34
GMAT 2: 740 Q50 V40
GPA: 4
Send PM
Re: Economist: Some policymakers believe that our country's continued econ [#permalink]
vineetgupta wrote:
Economist: Some policymakers believe that our country's continued economic growth requires a higher level of personal savings than we currently have. A recent legislative proposal would allow individuals to set up savings accounts in which interest earned would be exempt from taxes until money is withdrawn from the account. Backers of this proposal claim that its implementation would increase the amount of money available for banks to loan at a relatively small cost to the government in lost tax revenues. Yet, when similar tax-incentive programs were tried in the past, virtually all of the money invested through them was diverted from other personal savings, and the overall level of personal savings was unchanged.

The author criticizes the proposed tax-incentive program by


Policy makers are suggesting to increase the savings in accounts to increase the loan providing capability of banks.
However, previous implementation of the plan did not provide the required effect. Instead, the balance remained the same and the public transferred money from one account to the new account

(A) challenging a premise on which the proposal is based
- the policy makers propose a plan - provide interest free savings to increase the amount available for loan.
Premise - interest free savings will encourage public to save more money.
The author says that people will not save money; people will simple transfer the money
-correct

(B) pointing out a disagreement among policymakers
- the disagreement is between author and policymakers.
-Wrong

(C) demonstrating that the proposal's implementation is not feasible
- feasibility of proposal implementation is not discussed.
-Wrong

(D) questioning the judgment of the proposal's backers by citing past cases in which they had advocated programs that have proved ineffective
- judgement of proposal's backers is that public will deposit extra money.
- this judgement is questioned by using the past reference case.
-correct

(E) disputing the assumption that a program to encourage personal savings is needed
- this is out of context
-wrong

Posted from my mobile device
Intern
Intern
Joined: 05 Dec 2019
Posts: 5
Own Kudos [?]: 0 [0]
Given Kudos: 5
Send PM
Re: Economist: Some policymakers believe that our country's continued econ [#permalink]
For the confusion between A and D, I would like to mention that in option D "they" gives it all away. The people who are backing this claim now are not the same people who did it in the past. So, adding the pronoun "they" in D makes is evident that the answer should be A.
To consolidate A, the premise (higher savings levels being essential) leads to proposal (better economic conditions). The author attacks the very premise on which the proposal is based.
GMAT Club Verbal Expert
Joined: 13 Aug 2009
Status: GMAT/GRE/LSAT tutors
Posts: 6917
Own Kudos [?]: 63649 [1]
Given Kudos: 1773
Location: United States (CO)
GMAT 1: 780 Q51 V46
GMAT 2: 800 Q51 V51
GRE 1: Q170 V170

GRE 2: Q170 V170
Send PM
Re: Economist: Some policymakers believe that our country's continued econ [#permalink]
1
Kudos
Expert Reply
Youraisemeup wrote:
Even though OA is A I am confused here because the author is not challenging a premise... He is challenging an underlying assumption of policymakers that Tax incentives are enough to make people save more. nightblade354 GMATNinja sir please share your views..

The policymakers in the passage have an overall goal of continuing our country's economic growth. To accomplish this goal, they believe that people need a higher personal savings rate.

They will encourage this higher personal savings rate by implementing a proposal that "would allow individuals to set up savings accounts in which interest earned would be exempt from taxes until money is withdrawn from the account."

Their justification that the plan will have the intended effect is that "its implementation would increase the amount of money available for banks to loan at a relatively small cost to the government in lost tax revenues." In other words, the economy will continue to grow because banks will have more money to lend out.

The author criticizes this line of reasoning by saying that "when similar tax-incentive programs were tried in the past, virtually all of the money invested through them was diverted from other personal savings, and the overall level of personal savings was unchanged." This is directly attacking the justification listed above: if people just divert one kind of savings to another, banks will not have more money to lend out.

A premise is "a previous statement or proposition from which another is inferred or follows as a conclusion." Or, a premise is just a link in the logical chain that leads to an overall conclusion. An assumption can fill a similar role, but it is an unstated link in the logical chain.

Because the author challenges the stated assertion that the proposal "would increase the amount of money available for banks to loan," we can say that he/she is attacking a premise on which the proposal is based.

I hope that helps!
Manager
Manager
Joined: 27 Nov 2022
Posts: 84
Own Kudos [?]: 2 [0]
Given Kudos: 11
Send PM
Economist: Some policymakers believe that our country's continued econ [#permalink]
Clear answer A
Author is not using that incidence to criticize the proposal but rather use that incidence to prove why the idea as a whole does not work because the it is diverted by other personal savings.
So the catch is the policy tried before does not prove why it is a wrong strategy but why that policy failed being explained by the author certainly criticizes the premise of the whole concept.
That is why D) is not correct.
GMAT Club Bot
Economist: Some policymakers believe that our country's continued econ [#permalink]
Moderators:
GMAT Club Verbal Expert
6917 posts
GMAT Club Verbal Expert
238 posts
CR Forum Moderator
832 posts

Powered by phpBB © phpBB Group | Emoji artwork provided by EmojiOne